Complex Conjugate of a Function

Click For Summary
To find the complex conjugate of the function 1/(1-Ae^(i(a+b))), where A is real and a, b are complex functions, the correct expression is 1/(1-Ae^(-i(a^*+b^*))). The discussion emphasizes that while the property f(z^*) = f(z)^* does not generally hold for complex functions, it can apply if a and b are treated as mere complex numbers. A counterexample illustrates that some functions may not adhere to this property, highlighting the importance of the specific nature of a and b. Ultimately, if a and b are indeed just complex numbers, the initial assumption about the conjugate is valid. The conclusion confirms that the approach is correct under those conditions.
Niles
Messages
1,834
Reaction score
0

Homework Statement


Hi

I have a complex function of the form
<br /> \frac{1}{1-Ae^{i(a+b)}}<br />
I want to take the complex conjugate of this: The parameters a and b are complex functions themselves, but A is real. Am I allowed to simply say
<br /> \frac{1}{1-Ae^{-i(a^*+b^*)}}<br />
where * denotes the c.c.? I seem to vaguely remember that f^*(x) = f(x^*).
 
Physics news on Phys.org
Be careful; the rule f(z^*)=f(z)^* doesn't in general work when z is a complex number.
Consider the following counterexample: take f(z)=i|z|, where |\cdot | is the absolute value. Then f(z^*)=i|z^*|=i|z|=f(z) but f(z)^*=(i|z|)^*=-i|z|=-f(z), so we have f(z^*)\neq f(z)^*.

In general, your function f can be very nice (the example above isn't complex-differentiable) but still fail to have this property.

For your problem, it's going to depend a lot on what your functions a,b are.
 
Thanks for helping out. If a and b are just complex numbers, then it should be correct, no?
 
Niles said:
Thanks for helping out. If a and b are just complex numbers, then it should be correct, no?

Yes, if a and b are just complex numbers, then it works out like that.
 
Question: A clock's minute hand has length 4 and its hour hand has length 3. What is the distance between the tips at the moment when it is increasing most rapidly?(Putnam Exam Question) Answer: Making assumption that both the hands moves at constant angular velocities, the answer is ## \sqrt{7} .## But don't you think this assumption is somewhat doubtful and wrong?

Similar threads

  • · Replies 5 ·
Replies
5
Views
2K
Replies
5
Views
2K
  • · Replies 17 ·
Replies
17
Views
3K
  • · Replies 8 ·
Replies
8
Views
1K
Replies
3
Views
1K
  • · Replies 26 ·
Replies
26
Views
5K
  • · Replies 7 ·
Replies
7
Views
1K
Replies
7
Views
2K
  • · Replies 4 ·
Replies
4
Views
2K